Can this odd-looking fraction be integrated?

Click For Summary
The integral in question is ∫(dx/(a²+(x-1/x)²)), which presents challenges due to its unusual denominator. Attempts to simplify it using trigonometric substitutions or standard forms have not yielded results. A suggestion was made to expand and simplify the expression by multiplying by x²/x² and factoring the denominator. The discussion then shifted to treating the denominator as a quadratic equation by substituting variables, which could facilitate integration. Overall, the conversation focuses on finding a viable method to tackle the integration of this complex fraction.
Sleek
Messages
60
Reaction score
0

Homework Statement



\displaystyle \int{\frac{dx}{a^2+\left(x-\frac{1}{x} \right)^2}}

Homework Equations



-

The Attempt at a Solution



This one looks a bit odd. Had the denominator been a^2 + x^2, it is in one of the standard forms, whose integral is \frac{1}{a} \atan{\frac{x}{a}}. But the denominator is in the form of a^2 + u^2 (where u is a function of x). I did try some manipulations, but to no avail. I tried putting x as sin(theta), but got something like cos(theta)d(theta)/(a^2+cos^4(theta)/sin^2(theta)), which seems even more complex. If someone can just point me into the direction to look, I'll attempt the solution.

Thank you,
Sleek.
 
Physics news on Phys.org
Expand the brackets, simplify, multiply the entire integral by x^2/x^2, factor the denominator and partial fractions.
 
Thanks for the quick reply, I'm currently here,

\displaystyle \int{\frac{x^2dx}{x^2(a^2-2)+x^4+1}

I don't see how I can factorize/simplify the denominator or the expression...?

Regards,
Sleek.
 
Last edited:
Well let a^2-2 =b and u=x^2. Now it resembles a nice quadratic equation =]
 
Question: A clock's minute hand has length 4 and its hour hand has length 3. What is the distance between the tips at the moment when it is increasing most rapidly?(Putnam Exam Question) Answer: Making assumption that both the hands moves at constant angular velocities, the answer is ## \sqrt{7} .## But don't you think this assumption is somewhat doubtful and wrong?

Similar threads

  • · Replies 19 ·
Replies
19
Views
2K
Replies
3
Views
2K
  • · Replies 14 ·
Replies
14
Views
2K
  • · Replies 4 ·
Replies
4
Views
1K
  • · Replies 5 ·
Replies
5
Views
2K
  • · Replies 22 ·
Replies
22
Views
3K
  • · Replies 3 ·
Replies
3
Views
1K
Replies
28
Views
2K
  • · Replies 105 ·
4
Replies
105
Views
6K
  • · Replies 4 ·
Replies
4
Views
2K